Clinical Cardiology Multiple Choice Questions

Description

1 PA School Quiz on Clinical Cardiology Multiple Choice Questions, created by Elle Ashe on 10/02/2017.
Elle Ashe
Quiz by Elle Ashe, updated more than 1 year ago
Elle Ashe
Created by Elle Ashe about 7 years ago
1730
1

Resource summary

Question 1

Question
A 45 year old male patient presents with abrupt severe chest pain, flank pain, and syncope. Physical exam reveals asymmetric extremity blood pressure. Percussion of the lungs is dull and there is decreased tactile fremitus. On ausculatation you hear a decrescendo diastolic murmur best heard at the lower left sternal border. It has a blowing quality and is heard best when patient is leaning forward. CT with contrast shows a displaced trachea and pleural effusion. Which diagnosis is most likely?
Answer
  • Coarctation of the aorta
  • Thoracic aortic aneurysm
  • Aortic dissection
  • MI
  • PE

Question 2

Question
An 80 year old female presented to the emergency room with an MI. Prior medical history includes hypertension, and no prior history of angina. On the 3rd day EKG and physical exam showed abrupt pulseless electrical activity. Shortly after she died. What is the likely complication?
Answer
  • Free wall rupture
  • Ventricular septal defect
  • Papillary muscle rupture
  • Ventricular wall aneurysm
  • Congestive heart failure

Question 3

Question
What is the most common peripheral aneurysm?
Answer
  • Femoral artery
  • Popliteal artery
  • Carotid artery
  • Brachial artery

Question 4

Question
What is the most common cause of abdominal aortic aneurysm?
Answer
  • Marfan's syndrome
  • Bicuspid aortic valve
  • Atherosclerosis
  • Aortic stenosis
  • STEMI

Question 5

Question
A 35 year old female presents to the ED with dyspnea, hoarseness, hemoptysis, and lower leg edema. Clinical examination shows a loud S1 with an opening snap and rumbling mid-diastolic murmur. EKG shows left atrial enlargement and right ventricular hypertrophy. Chest x ray confirms this and shows interstitial edema. What is the likely diagnosis?
Answer
  • Bacterial endocarditis
  • Tricuspid stenosis
  • Mitral regurgitation
  • Aortic stenosis
  • Mitral stenosis

Question 6

Question
What can cause a decreased amplitude of P-waves, increase in PR interval, bradycardia in the form of AV block, and even an absence of P waves eventually leading to a "sine wave."
Answer
  • Hypokalemia
  • Hyperkalemia
  • Hypocalcemia
  • Hypercalcemia

Question 7

Question
Which of the following is the most common cause of multifocal atrial tachycardia?
Answer
  • Fever
  • MI
  • Valve disease
  • Lung disease
  • Low Mg/K

Question 8

Question
Which of the following ventricular septal defects can you expect to close spontaneously more so than the others?
Answer
  • Perimembranous
  • Muscular
  • Inlet
  • Supracristal

Question 9

Question
An EKG shows a U wave which occurs just after the T-wave and is of smaller amplitude than the T-wave. Further, there is flattening of the T-wave and ST-depression. What does this indicate?
Answer
  • Hypokalemia
  • Hyperkalemia
  • Hypocalcemia
  • Hypercalcemia

Question 10

Question
High uric acid and glucose intolerance are typical of which type of lipid disorder according to Fredrickson classification?
Answer
  • Type I
  • Type IIa
  • Type IIb
  • Type III

Question 11

Question
Electrical altercans is a finding on EKG of alternating amplitude of QRS complexes. Which of these diseases is most associated with this finding?
Answer
  • Atrial septal defect
  • Patent ductus arteriosus
  • Pericarditis
  • Myocarditis
  • NSTEMI

Question 12

Question
Which of the following is NOT a component of long term medical management of MI?
Answer
  • Beta blockers
  • Statins
  • Aspirin
  • Calcium channel blockers
  • P2Y12 blockers
  • ACEi

Question 13

Question
What is the greatest source of the biomarker CPK?
Answer
  • Skeletal muscle
  • Myocardial
  • Brain
  • Lungs
  • Kidneys

Question 14

Question
What pathology causes the greatest absolute number of ascending (thoracic) aortic dissections?
Answer
  • Marfan's Symdrome
  • Bicuspid aortic valve
  • Coarctation of the aorta
  • Mitral valve stenosis

Question 15

Question
A 70 year old female presents with ascites, edema, and jaundice. Physical exam shows JVD and hepatomegaly. Auscultation reveals a decrescendo diastolic rumble at the lower left sternal border that is louder with inspiration. What is the likely diagnosis?
Answer
  • Mitral stenosis
  • Mitral regurgitation
  • Mitral valve prolapse
  • Tricuspid stenosis
  • tricuspid regurgitation

Question 16

Question
A shortened QT interval, shortened ST segment, and Osborn waves (J waves) can be caused by?
Answer
  • Hypokalemia
  • Hyperkalemia
  • Hypocalcemia
  • Hypercalcemia

Question 17

Question
Which of the following is the most common ventricular septal defect?
Answer
  • Perimembranous
  • Muscular
  • Inlet/Posterior
  • Supracristal

Question 18

Question
Which is the most common form of lipid disorder according to the Fredrickson Lipid Classification?
Answer
  • Type I
  • Type IIa
  • Type IIb
  • Type III
  • Type IV
  • Type V

Question 19

Question
Peaked T waves best seen in the precordial leads, widening of QRS complexes, and sometimes ST depression may indicate.
Answer
  • Hypokalemia
  • Hyperkalemia
  • Hypocalcemia
  • Hypercalcemia

Question 20

Question
A 60 year old male with a prior medical history of pulmonary hypertension presents with ascites and lower extremity edema. Physical exam reveals an elevated JVP and a large pulsatile liver. Auscultation shows a holosystolic murmur at lower left sternal border that is louder with inspiration. Which of the following is most likely
Answer
  • Mitral regurgitation
  • Mitral valve prolapse
  • Aortic stenosis
  • Aortic regurgitation
  • Tricuspid regurgitation

Question 21

Question
A posterior wall MI involves which artery?
Answer
  • RCA
  • LAD
  • Circumflex
  • RCA and Circumflex
  • LAD and RCA
  • LAD and circumflex

Question 22

Question
What is the most common etiology of mitral stenosis?
Answer
  • Pulmonary hypertension
  • Infective endocarditis
  • Mitral annular calcification
  • Rheumatic heart disease

Question 23

Question
Which of the following would be first line treatment for acute AV node Reentry Tachycardia?
Answer
  • Adenosine
  • Lidocain
  • Amiodarone
  • Digoxin
  • Atropine

Question 24

Question
A 60 year old male complains of a cramping sensation in his calf after walking 50 feet or so. When he stops, the pain goes away. The pain returns upon walking. What term best describes these symptoms?
Answer
  • Thromangitis obliterans
  • Claudication
  • Collateralization
  • Compartment syndrome

Question 25

Question
What is the most common visceral aneurysm?
Answer
  • Splenic
  • Celiac
  • SMA
  • IMA
  • Renal

Question 26

Question
What is the standard antimicrobial therapy for bacterial endocarditis prophylaxis?
Answer
  • Penicillin
  • Amoxicillin
  • Clindamycin
  • Azithromycin
  • Ceftriaxone

Question 27

Question
A 35 year old patient recovering from a cold presents with dyspnea, orthopnea and edema. Physical exam shows elevated JVP, peripheral edema, and crackles. Auscultation reveals a displaced PMI and an S3 sound. EKG shows ST elevation. Chest x ray shows cardiomegaly, pulmonary congestion and thickened sub-plueral interlobular septa (Kerley B lines). Which of these diagnoses is most reasonable?
Answer
  • STEMI
  • Pericarditis
  • Constrictive pericarditis
  • Coarctation of the aorta
  • Myocarditis

Question 28

Question
Peripheral neuropathy is most often associated with which type of lipid disorder?
Answer
  • Type I
  • Type IIa
  • Type IIb
  • Type III
  • Type IV
  • Type V

Question 29

Question
A patient with a history of TB presents with peripheral edema, and ascites. Physical exam reveals hepatomegaly, elevated JVP. Kussmaul's sign (Increased JVP on inspiration) is positive but no pulsus paradoxus is noted. On auscultation a high pitched heart sound is noted during diastole. Which of the following can most reasonably explain these findings
Answer
  • Hypertrophic cardiomyopathy
  • Dilated cardiomyopathy
  • Myocarditis
  • Cardiac tamponade
  • Constrictive pericarditis

Question 30

Question
A patient presents with weakness, dyspnea and pulmonary edema. Auscultation reveals a blowing holosystolic murmur, immediately after S1 that radiates to the axilla. Based off this information alone, which of the following is most likely
Answer
  • Mitral stenosis
  • Mitral regurgitation
  • Mitral valve prolapse
  • Aortic stenosis
  • Aortic regurgitation

Question 31

Question
Which of the following presentations of cardiomyopathy is most common?
Answer
  • Ischemic dilated cardiomyopathy
  • Non-ischemic dilated cardiomyopathy
  • Hypertrophic non-dilated cardiomyopathy
  • Restrictive/Infiltrative cardiomyopathy

Question 32

Question
Which of the following Fredrickson Lipid Classification categories involved familial lipoprotein lipase (LPL) deficiency resulting in high triglycerides and chylomicrons. Signs include pancreatitis, lipemia retinalis, hepatosplenomegaly and eruptive xanthomas.
Answer
  • Type I
  • Type III
  • Type IV
  • Type V

Question 33

Question
A lateral wall MI involves which artery?
Answer
  • RCA
  • LAD
  • Circumflex
  • LAD and RCA
  • LAD and circumflex

Question 34

Question
Which of the following tests is the gold standard for diagnosing mitral regurgitation?
Answer
  • CXR
  • ECG
  • ECHO
  • Cardiac catheterization
  • Magnetic resonance angiogram

Question 35

Question
A prominent S4 and a crescendo-decrescendo murmur at LLSB (similar to AS but not as RUSB)( that does not radiate to the carotids and increases on expiration (more blood fills LV) would make you most concerned for?
Answer
  • Dilated Cardiomyopathy
  • Hypertrophic cardiomyopathy
  • Restrictive cardiomyopathy
  • Pericarditis
  • Myocarditis

Question 36

Question
A patient presents with fatigue, dyspnea and chest pain. EKG shows diffuse ST elevation. He is hypotensive with pressure varying from inspiration to expiration. In addition, physical exam reveals muffled heart sounds and elevated JVP that increases on inspiration. Chest x ray shows cardiomegaly resembling a water-bottle. Which of the following is the most likely diagnosis?
Answer
  • Constrictive pericarditis
  • Cardiac tamponade
  • STEMI
  • Prinzmetal's angina
  • Myocarditis

Question 37

Question
In western countries, what is the most common valvular heart disease?
Answer
  • Mitral stenosis
  • Tricuspid stenosis
  • Aortic stenosis
  • Pulmonic stenosis

Question 38

Question
Premature atherosclerosis, corneal arcus and tendon xanthomas are signs of which type of lipid disorder?
Answer
  • Type I
  • Type IIa
  • Type IIb
  • Type III

Question 39

Question
What is the most common cause of valvular heart disease in developing countries
Answer
  • Mitral stenosis
  • Mitral valve prolapse
  • Tricuspid stenosis
  • Tricuspid regurgitation
  • Aortic stenosis

Question 40

Question
Which class of drugs is most commonly indicated for treatment of hypertrophic cardiomyopathy?
Answer
  • Statins
  • Beta blockers
  • Diuretics
  • ACEi

Question 41

Question
A mid-systolic click followed by a late systolic murmur is virtually diagnostic for which of the following diseases?
Answer
  • Mitral valve stenosis
  • Mitral valve regurgitation
  • Mitral valve prolapse
  • Aortic stenosis
  • Aortic regurgitation

Question 42

Question
Which of the following is the most common etiology of tricuspid regurgitation?
Answer
  • Trauma
  • Congenital abnormalities
  • Right ventricular dilation
  • Bacterial endocarditis

Question 43

Question
A 60 year old male smoker with COPD and hypertension presents with abdominal pain and lower back pain. Physical exam reveals a pulsatile abdomen and "blue toes." Which of the following tests is the gold standard for evaluating and diagnosing this pathology?
Answer
  • EKG
  • Ultrasound
  • CXR
  • CT
  • MRI

Question 44

Question
Which of the following complications from acute MI is the most common cause of mortality
Answer
  • Arrhythmia
  • Congestive heart failure
  • Recurrent infarction
  • Septal rupture
  • Ventricular wall aneurysm

Question 45

Question
Which of the following is the most common congenital heart defect?
Answer
  • Patent ductus arteriosus
  • Atrial septal defect
  • Ventricular septal defect
  • Patent ductus venosum

Question 46

Question
A 58 year old female presents with dyspnea, angina and multiple cases of syncope. Auscultation reveals a systolic, high pitched, crescendo-decrescendo murmur that radiates to the carotids. Further, pulusus parvus and tardus is noted. What is the most likely diagnosis?
Answer
  • Aortic stenosis
  • Pulmonic stenosis
  • Tricuspid stenosis
  • Mitral stenosis

Question 47

Question
In an aneurysm below the inguinal ligament, what is the most common complication?
Answer
  • Rupture
  • Embolization
  • Thrombosis
  • Dissection

Question 48

Question
Which of the following would NOT be an indication for bacterial endocarditis prophylaxis?
Answer
  • Prosthetic heart valves
  • Hypertrophic cardiomyopathy
  • Prior history of infective endocarditis
  • Complex cyanotic congenital heart disease

Question 49

Question
If you suspect a patient with HIV has non-ischemic dilated cardiomyopathy myocarditis, what test would best confirm your diagnosis?
Answer
  • TEE
  • EKG
  • Heart biopsy
  • CT scan with contrast
  • Magnetic resonance angiogram

Question 50

Question
Operating on an AAA is indicated in an otherwise healthy individual when it gets larger than _______?
Answer
  • 5cm
  • 5.5cm
  • 6cm
  • 6.5cm
  • 7cm

Question 51

Question
What is standard risk factor reduction therapy for asymptomatic carotid disease?
Answer
  • Aspirin, Statin, ACEi
  • Coumadin, Statin
  • Beta blocker, Statin, ACEi
  • Loop diuretics, Statin, ACEi

Question 52

Question
Which of the following arrhythmias would NOT present with a narrow QRS and irregular rhythm?
Answer
  • Afib
  • MAT
  • Aflutter
  • VT

Question 53

Question
Carotid endarterectomy surgery is indicated at what level of stenosis?
Answer
  • 50%
  • 60%
  • 70%
  • 80%
  • 90%

Question 54

Question
A patient with CHF presents for further work up. Chest x ray shows cardiomegaly, pulmonary edema and rib notching. Which of the following would be a reasonable cause of his CHF?
Answer
  • Hypoplastic left heart syndrome
  • Aortic valve stenosis
  • Myocarditis
  • Cardiac tamponade
  • Coarctation of the aorta

Question 55

Question
A 39 year old male presents with claudication, Raynaud's phenomenon and digital ulceration. Which of the following would be most reasonable?
Answer
  • Acute arterial occlusion
  • Thromboangitis obliterans (Buerger Disease)
  • DVT
  • Chronic venous insufficiency
  • Abdominal aortic aneurysm

Question 56

Question
An inferior wall MI involves which artery?
Answer
  • RCA
  • LAD
  • Circumflex
  • RCA and circumflex
  • LAD and RCA
  • LAD and circumflex

Question 57

Question
A patient comes to the clinic on follow up from an acute ST segment elevation myocardial infarction (STEMI). Examination reveals a new systolic heart murmur. Which of the following is the best explanation for this finding?
Answer
  • Acute aortic stenosis from infarction
  • Acute tricuspid regurgitation from infarction
  • Acute mitral regurgitation from infarction
  • Acute mitral stenosis from infarction

Question 58

Question
A 53 yo healthy male undergoes an exercise stress test, running on a treadmill until a maximum exertion is obtained. Which of the following statements correctly describes the effects of autonomic nerve activity on the cardiovascular system in this healthy individual?
Answer
  • Inhibition of parasympathetic nerves decreases total peripheral resistance
  • Inhibition of parasympathetic nerves increases heart rate
  • Inhibition of parasympathetic nerves increases total peripheral resistance
  • Stimulation of parasympathetic nerves decreases the strength of cardiac contractions
  • Stimulation of sympathetic nerves decreases the strength of cardiac contractions

Question 59

Question
A three year old female presents for a well-child visit and was found to have rough, continuous murmur hear over the left pulmonary area. What is this murmur?
Answer
  • Aortic stenosis
  • Patent ductus arteriosis
  • Mitral regurgitation
  • Pulmonic stenosis

Question 60

Question
A 21 year old woman presents to the clinic with complaints of palpitations and a headache. On physical exam, the patient is anxious and diaphoretic with a blood pressure of 175/105 mm Hg and a heart rate of 122 b[m. ECG demonstrates sinus tachycardia. Based on this presentation, what is a likely diagnosis.
Answer
  • Supraventricular tachycardia
  • Acute coronary syndrome
  • Aortic dissection
  • Pheochromocytoma

Question 61

Question
Which of the following radiographic findings is associated with tetralogy of Fallot?
Answer
  • Decrease in right ventricular size
  • Rib notching
  • Increased pulmonary vascular markings
  • Boot shaped heart

Question 62

Question
What is the cardiac ausculatory hallmark of an atrial septal defect?
Answer
  • Paradoxical split S2
  • Wide, fixed split S2
  • Systolic ejection murmur second right intercostal space
  • Holosystolic murmur heard a apex that radiates to axilla

Question 63

Question
A 53 year old man with history of tobacco presents to the emergency department complaining of severe, ripping, substernal chest pain that radiates to his back. His blood pressure is 175/92 mm Hg. Chest X-ray reveals a widened mediastinum, What is his most likely diagnosis.
Answer
  • Acute coronary syndrome
  • Pneumothorax
  • Aortic dissection
  • Mitral stenosis

Question 64

Question
An 81 year old woman with a 10 year history of well-controlled atrial fibrillation complains of a 6-day history of fatigue, dyspnea, and a 10 pound weight gain. She denies angina or diaphoresis. Based on this history what is the most likely diagnosis?
Answer
  • Acute coronary syndrome
  • Heart failure
  • Sinus tachycardia
  • Cardiac tamponade

Question 65

Question
A 67 year old male with history of coronary artery disease with prior myocardial infarction and stable, chronic heart failure returns tot he clinic for follow up. Which of the following medication regimens is most appropriate for him?
Answer
  • Aspirin, metoprolol, valsartan, clopidogrel, morphine, diuretic
  • Heparin, atenolol, atorvastatin, clonidine, nitroglycerine
  • Amlodipine, Lisinopril, aspirin, valsartan, atorvastatin, diuretic, metoprolol
  • Aspirin, metoprolol, Lisinopril, atorvastatin, diuretic, nitroglycerin

Question 66

Question
A 37 year old diabetic patient with history of paroxysmal atrial fibrillation and hypertension presents with sudden onset of severe pain in his left arm. On physical examination, the left arm is cool and pale with nonpalpable radial and ulnar pulses. What is the most likely cause for his pain?
Answer
  • Atherosclerotic peripheral vascular disease
  • Venous stasis disease
  • Raynaud syndrome
  • Arterial embolization

Question 67

Question
Which of the following is the most likely cause of paradoxical splitting of S2?
Answer
  • Pulmonic stenosis
  • Left bundle branch block
  • Atrial septal defect
  • Right ventricular failure

Question 68

Question
Orthostatic hypotension is defined as a drop in systolic blood pressure of at least ____________ or a drop of diastolic blood pressure of at least _________________ within 3 minutes of standing from the sitting position.
Answer
  • 10 mm Hg; 20 mm Hg
  • 10 mm Hg; 5 mm Hg
  • 20 mm Hg; 10 mm Hg
  • 5 mm Hg; 10 mm Hg

Question 69

Question
A 47 year old male presents with shortness of breath. On physical examination, the patient is sitting upright with moderately labored breathing, marked jugular venous distention (JVD), bilateral pulmonary rales, and a S3 gallop. Blood pressure is 120/90 and heart rate is 110 bpm. Chest x-ray shows pulmonary congestion. ECG demonstrates sinus tachycardia with low voltage QRS. Which class of medication should be given to this patient immediately to improve symptoms.
Answer
  • Ace inhibitors
  • Beta blockers
  • Calcium channel blockers
  • Diuretics

Question 70

Question
Which of the following medications is used to control the ventricular rate during an acute event of rapid atrial fibrillation?
Answer
  • Beta blocker
  • Digoxin
  • Warfarin
  • Nitroglycerin

Question 71

Question
Which of the following is a contraindication for an exercise treadmill stress test.
Answer
  • Mild aortic stenosis
  • Unstable angina
  • Previous myocardial infarction
  • Nonspecific ECG changes

Question 72

Question
Which of the following antiarrhythmic drugs most potently blocks sodium channel current in the myocardium?
Answer
  • Flecainide
  • Procainamide
  • Amiodarone
  • Lidocaine

Question 73

Question
According to the American Diabetes Association, risk reduction for cardiovascular events in patients with diabetes can be achieved by the use of:
Answer
  • Beta blockers
  • Statins
  • Cholesterol absorption inhibitors
  • Aldosterone

Question 74

Question
A 58 year old female with history of Type 2 diabetes mellitus presents for routine follow up. Which of the following would have the LEAST impact on prevention of her risk of developing macrovascular complications from diabetes?
Answer
  • Strict control of blood glucose levels
  • HMG-CoA reductase inhibitor therapy
  • Estrogen/progestin combination therapy
  • ACEi to control her hypertension

Question 75

Question
Secondary [prevention of an St-elevation myocardial infarction should include which of the following measures?
Answer
  • Estrogen therapy
  • ACEi
  • Folate or Vitamin B6
  • Calcium channel blockers

Question 76

Question
An increase in which of the following factors is most likely to increase the preload?
Answer
  • Arterial vascular tone
  • Stroke volume
  • Heart rate
  • Intravascular volume

Question 77

Question
Which is the mechanism by which cardiac tamponade impedes cardiac output?
Answer
  • Decreased inflow of blood to ventricles
  • Decreased pressure in pericardium
  • Increased pressure decreases sinus rhythm
  • Increase in end-diastolic volume

Question 78

Question
Which of the following is used to describe diastolic dysfunction?
Answer
  • Increased afterload
  • Decreased preload
  • Decreased stroke volume
  • Decreased ventricular myocardial compliance

Question 79

Question
A 37 year old woman with a history of hypertension presents to the emergency department with complaints of shortness of breath. She was placed on a telemetry monitor. What is your diagnosis?
Answer
  • Normal sinus rhythm
  • Atrial fibrillation
  • Sinus tachycardia
  • Nodal rhythm

Question 80

Question
A 29 year old man presents with extreme fatigue and shortness of breath. What is the abnormality noted in the ECG?
Answer
  • Complete AV Block
  • Type II second degree AV block
  • Junctional rhythm
  • Atrial fibrillation

Question 81

Question
You have started your patient who has giant cell arteritis on her glucocorticoid therapy. Which of the following laboratory studies would be used to monitor her response to this therapy?
Answer
  • Cardiac enzymes
  • Sedimentation rate
  • Rheumatoid factor
  • Complete blood cell count

Question 82

Question
A 50 year old woman presents to the office with a 1 week history of substernal chest pain. The symptoms occur with climbing the stairs in her home and with moderate-paced walking. The pain is relieved with rest, She denies any current chest pain. Her physical examination and ECG are normal. What I the nest test you would order?
Answer
  • Exercise treadmill stress test
  • Cardiac enzymes
  • Radionuclide stress myocardial perfusion imaging
  • Echocardiogram
Show full summary Hide full summary

Similar

The Endocrine System
DrABC
Respiratory anatomy
James Murdoch
Fluid and Electrolyte Imbalances
D R
Myalgic Encyphalomyelitis
Joanna Rogowska
Epidemiology
Danielle Richardson
History of Medicine: Ancient Ideas
James McConnell
Epithelial tissue
Morgan Morgan
4. The Skeletal System - bones of the skull
t.whittingham
Neuro anatomy
James Murdoch
Basic Immunology Principles
Robyn Hokulani-C